LSAT and Law School Admissions Forum

Get expert LSAT preparation and law school admissions advice from PowerScore Test Preparation.

 lsatstudier
  • Posts: 49
  • Joined: Oct 24, 2016
|
#33392
Hi,

I was wondering if someone could explain how to go about selecting answer choice D. I initially chose A, thinking that such information would weaken the fact that the glacier could not move the boulder that far. The second time around I chose C, even though answer choices B and C look very similar. It makes sense after looking at it for awhile that D provides an alternative explanation. However, I am not convinced of my explanation.

Thank you!
 Charlie Melman
PowerScore Staff
  • PowerScore Staff
  • Posts: 85
  • Joined: Feb 10, 2017
|
#33428
Hi L,

This is a classic LSAT stimulus, in that it gives one possible explanation for a set of facts, but does not consider any of an infinite set of alternative explanations. The correct answer choice is going to give us a fact or explanation that highlights this.

Answer choice (A): "Most" on the LSAT means that the thing in question has a 51-100% chance of occurring. Even if what answer choice (A) says is true, it could be that 49% of glaciers are moved hundreds of miles. That's barely a weakener.

Answer choice (B): The stimulus says the rock was moved hundreds of miles. If the closest source of rock is 50 miles away, then it's entirely possible that there could be a farther source that's hundreds of miles away.

Answer choice (C): Same as answer choice (B). The south/north distinction does not matter, since both (C) and (B) could be true while the stimulus is true.

Answer choice (D): This is correct. If the glaciers were moving southward to the site mentioned in the stimulus, they had to come from north of the site. If there are no sources of volcanic rock north of the site, then the "geological birthplace" of the rock couldn't have been north of the site, and thus the glacier couldn't have carried it down.

Answer choice (E): Irrelevant. We don't care about other volcanic boulders, we care about the geological origin of the boulder in question.

Hope this makes sense!
 bk1111
  • Posts: 103
  • Joined: Apr 22, 2017
|
#37176
Hi, I am trying to make a good prephrase for this question, but I am not sure how to go about it. I came up with something along the lines of "It is not necessarily true that the boulder deposited here by a glacier." The answer choices all sounded very similar to me. In general, I have a harder time with prephrases for science-related topics. Do you have any advice about how to attack such questions?
 Adam Tyson
PowerScore Staff
  • PowerScore Staff
  • Posts: 5153
  • Joined: Apr 14, 2011
|
#37526
I think your prephrase is on the right track, bk1111. Since the conclusion is that a glacier deposited the boulder, you want to pick an answer that calls that into question. Maybe the boulder got here some other way? If we had an answer that tracked with that, we'd be happy - something like "volcanoes often eject boulders with such force that they land far away from their source" or "this boulder shows markings on it consistent with its having been moved here by machines made by humans", or even "this boulder's mineral composition shows that it originated on another planet and could have landed on this spot from outer space".

Unfortunately, none of our answer choices suggests any alternate scenarios like those (Alternate causes, perhaps? Might this stimulus have causal reasoning?) so we have to look elsewhere.

Consider that most arguments on the LSAT are flawed, and that all flaws can be described one way or another as being based on a possibly flawed assumption. When we weaken a question, we are challenging the author's assumptions. When we strengthen, we are supporting those assumptions. So what assumption did our author make here? in order to claim that the volcanic boulder arrived at this spot by southward glacial movement, he must have assumed that somewhere to the north of this spot is a source of volcanic rock! To weaken the claim, attack that assumption - look for something that says "oh no there isn't!" If there is no source north of here, then his claim that a southward-moving glacier brought it here isn't just weakened, it's pretty well destroyed. There's answer D giving us exactly that.

Answer A sure does look like it would weaken the argument to me, but not by much, as our explanation says. Just because most boulders don't get moved hundreds of miles from their point of origin, that doesn't mean that all don't, or that this one didn't. I kept this as a contender, but only until something that did more to weaken the argument came along, and D fit the bill by making the claim not just a little weaker, but nearly impossible.

In prephrasing, consider those two approaches, which are very similar to the way we look at Assumption questions (Supporters and Defenders). On the one hand, consider alternatives to the claimed relationship, and on the other, consider attacking the stated claim directly by making it less likely to be viable.

Keep at it, you'll get there!
 brcibake
  • Posts: 55
  • Joined: Jul 19, 2017
|
#39770
Am I misunderstanding the stimulus? D doesn't make sense to me. If the glaciers were moving south, then to weaken it wouldn't you say there are no glaciers south?
Thank you
 Eric Ockert
PowerScore Staff
  • PowerScore Staff
  • Posts: 164
  • Joined: Sep 28, 2011
|
#39806
Well, the glaciers are moving south, which implies they are coming from the north. So, if southward moving glaciers are supposed to be bringing this volcanic rock to its current position, then the source of that volcanic rock would presumably be north of its current location. So if answer choice (D) is true, and there are no geological sources of volcanic rock north of the current location, that would make it very unlikely that glaciers were able to pick this rock up and bring it to its current position.

Hope that helps!
 beeryslurs
  • Posts: 11
  • Joined: Mar 18, 2020
|
#76612
Hi, I have a general question. What's the difference between questions that ask us to "weaken the argument" and those that ask us to "weaken the conclusion" (as in this one.) Should we attack them differently?
 Adam Tyson
PowerScore Staff
  • PowerScore Staff
  • Posts: 5153
  • Joined: Apr 14, 2011
|
#76995
Not really, beeryslurs, because we generally weaken a conclusion by attacking the reasoning that led to it. In other words, by weakening the argument. But we can weaken an argument either by showing that the premises do not support the conclusion (the conclusion might be true, but the evidence doesn't get you there), or by providing new information that directly questions the conclusion (suggesting that it is not true in spite of premises that might appear to support it). So for any weaken question, regardless of whether they say to weaken the argument or weaken the conclusion, you should be willing to consider either of those approaches. They both boil down to the same thing, and that is to cast doubt on what the author was trying to prove.
User avatar
 goingslow
  • Posts: 52
  • Joined: Aug 24, 2021
|
#93465
Hi there! My issue with answer choice (D) is that it describes the current geological source in the north, and yet the topic in the discussion was way back in history.

My thought process is: It could be that CURRENTLY there are no geological sources of volcanic rock north of this boulder, but there were such sources during the LAST ICE AGE, so what (D) describes as the current state of affairs is kind of irrelevant to what happened back then.

I'd appreciate any thought you may have! Thank you.
 Adam Tyson
PowerScore Staff
  • PowerScore Staff
  • Posts: 5153
  • Joined: Apr 14, 2011
|
#93480
That might be a problem for answer D IF we were looking for an answer that absolutely disproved the conclusion, goingslow. But since we only need to weaken the conclusion, we only need the answer to raise some amount of doubt. Think of it as the answer going in the negative column, weighing against the conclusion even if only a little.

Put another way, if you had made the original argument, and I came along and gave answer D to argue against you, you couldn't just ignore it or say "so what" in response. You would have to come back with "okay, maybe not right now, but in the past there could have been, like some volcano that existed prior to the last ice age but has since collapsed in upon itself." You would have to defend your position, because answer D attacks it. That's what makes it the right answer!

Get the most out of your LSAT Prep Plus subscription.

Analyze and track your performance with our Testing and Analytics Package.